Finding residues with Laurent series.

Click For Summary
The discussion revolves around finding the residue of the function f(z) = (4z - 6)/(z(2 - z)) at z = 0 using Laurent series. The user attempts to compute the series but arrives at an incorrect result of -6, while the solutions manual states the residue is -3. Key issues identified include incorrect equalities in the series expansion, particularly in the term involving 6/z(2 - z). The user is advised to re-evaluate their series expansions for accuracy. Correcting these errors is essential for obtaining the right residue value.
Terrell
Messages
316
Reaction score
26

Homework Statement


Use an appropriate Laurent series to find the indicated residue for ##f(z)=\frac{4z-6}{z(2-z)}## ; ##\operatorname{Res}(f(z),0)##

Homework Equations


n/a

The Attempt at a Solution


Computations are done such that ##0 \lt \vert z\vert \lt 2##.
##\frac{4z}{z(z-2)}=\frac{2}{1-z/2}## and ##\frac{6}{z(z-2)}=\frac{6}{z}\frac{1}{1-z/2}##.
\begin{align}
\frac{4z}{z(2-z)}=2\sum_{k=0}^{\infty}(\frac{z}{2})^k=2[1+\frac{z}{2}+\frac{z^2}{4}+\frac{z^3}{8}+\cdots]=2+z+\frac{z^2}{2}+\frac{z^3}{4}\\
\frac{6}{z}\frac{1}{1-z/2}=\frac{6}{z}\sum_{k=0}^{\infty}(\frac{z}{2})^k=\frac{6}{z}[1+\frac{z}{2}+\frac{z^2}{4}+\frac{z^3}{8}+\cdots]=\frac{6}{z}+3+\frac{3}{2}z+\frac{3}{4}z^2\\
f(z)=\frac{4z}{z(2-z)}-\frac{6}{z(2-z)}=-\frac{6}{z}-1-\frac{z}{2}-\frac{1}{4}z^2-\cdots
\end{align}
What am I doing wrong? The solutions manual gave an answer of -3 while according to my solution, it must be -6.
 
Physics news on Phys.org
Terrell said:
##\frac{4z}{z(z-2)}=\frac{2}{1-z/2}## and ##\frac{6}{z(z-2)}=\frac{6}{z}\frac{1}{1-z/2}##

Neither of these equalities is correct. The second "equality" is particularly problematic.
 
  • Like
Likes Terrell
Question: A clock's minute hand has length 4 and its hour hand has length 3. What is the distance between the tips at the moment when it is increasing most rapidly?(Putnam Exam Question) Answer: Making assumption that both the hands moves at constant angular velocities, the answer is ## \sqrt{7} .## But don't you think this assumption is somewhat doubtful and wrong?

Similar threads

  • · Replies 7 ·
Replies
7
Views
2K
  • · Replies 3 ·
Replies
3
Views
1K
  • · Replies 2 ·
Replies
2
Views
992
  • · Replies 9 ·
Replies
9
Views
2K
  • · Replies 3 ·
Replies
3
Views
2K
  • · Replies 10 ·
Replies
10
Views
2K
  • · Replies 14 ·
Replies
14
Views
3K
Replies
12
Views
3K
  • · Replies 7 ·
Replies
7
Views
1K
  • · Replies 12 ·
Replies
12
Views
5K